LSAT and Law School Admissions Forum

Get expert LSAT preparation and law school admissions advice from PowerScore Test Preparation.

User avatar
 Dave Killoran
PowerScore Staff
  • PowerScore Staff
  • Posts: 5852
  • Joined: Mar 25, 2011
|
#80542
This game is also discussed in our Podcast: LSAT Podcast Episode 70: The May 2020 LSAT-Flex Logic Games Section

Complete Question Explanation
(The complete setup for this game can be found here: https://forum.powerscore.com/lsat/viewtopic.php?t=33051)

The correct answer choice is (E).

If O hosts two meetings, then O must host a meeting in each semester:

  • ..... ..... ____ ..... ..... .....       ____

    ..... ..... _O_ ..... ..... .....       _O_

    ..... ..... _M/V_ ..... ..... ..... _M/V_
    ..... ..... Fall ..... ..... .....      Spring
With O being doubled, it is now the only city that can appear twice, and thus we know from the numerical distribution that M and V each can host only once (as is true for H and T as well). Thus, only two possible scenarios exist:


Scenario 1: M hosts in the fall, V hosts in the spring:

H and T are the two remaining cities that still must host, and when V hosts in the spring, T must host in the spring as well. H is then forced to host in the fall:

  • ..... ..... _H_ ..... ..... ..... _T_

    ..... ..... _O_ ..... ..... ..... _O_

    ..... ..... _M_ ..... ..... ..... _V_
    ..... ..... Fall ..... ..... ..... Spring

Scenario 2: V hosts in the fall, M hosts in the spring:

H and T are the two remaining cities that still must host, and when M does not host in the fall, via the contrapositive of the first rule H cannot host in the fall and thus must host in the spring. T is then forced to host in the fall:

  • ..... ..... _T_ ..... ..... ..... _H_

    ..... ..... _O_ ..... ..... ..... _O_

    ..... ..... _V_ ..... ..... ..... _M_
    ..... ..... Fall ..... ..... ..... Spring


Answer choice (A): Under scenario #2, H does not have to host a meeting in the fall, and thus this answer choice is incorrect.

Answer choice (B): Under scenario #1, T does not have to host a meeting in the fall, and thus this answer choice is incorrect.

Answer choice (C): Under scenario #1, M does not have to host a meeting in the spring, and thus this answer choice is incorrect.

Answer choice (D): In each scenario H and V do not host meetings in the same semester, and thus this answer choice is incorrect.

Answer choice (E): This is the correct answer choice. Both scenarios show T and V hosting meetings in the same semester.

Get the most out of your LSAT Prep Plus subscription.

Analyze and track your performance with our Testing and Analytics Package.